Domanda:
Cosa succede a livello quantistico quando una molecola ruota in luce polarizzata piana?
Joe
2017-10-11 08:41:06 UTC
view on stackexchange narkive permalink

Cosa succede a livello quantistico quando una molecola ruota una luce polarizzata piana?

Inoltre, perché gli enantiomeri ruotano la luce in direzioni opposte? Penserei che le onde elettromagnetiche interagirebbero con i campi elettrici e magnetici transitori all'interno della molecola stessa mentre si propaga e quindi la luce cambia direzione mentre passa attraverso la molecola. Tuttavia con questa idea sembra esserci un problema con il fatto che tutte le molecole non sarebbero orientate nelle stesse direzioni e quindi disperderebbero la luce in tutte le direzioni piuttosto che una rotazione netta in una direzione della luce. Sto cercando una spiegazione usando l'elettromagnetismo se può essere spiegato a quel livello o se non può, anche la risposta meccanica quantistica andrebbe bene.

Possibile duplicato di [Chiralità molecolare e rotazione ottica] (https://chemistry.stackexchange.com/questions/6271/molecular-chirality-and-optical-rotation)
Questo sicuramente non è un duplicato.
Sono d'accordo con @pentavalentcarbon. Il post Mithoron collegato è relativo a una parte della tua domanda (perché l'orientamento casuale di una soluzione non annulla la rotazione netta), ma non affronta il meccanismo fisico con cui avviene la rotazione ottica.
Non sono sicuro se conta come un duplicato se è su un SE diverso, ma vedi [come fanno i composti otticamente attivi a ruotare la luce polarizzata piana] (http://physics.stackexchange.com/questions/15503/how-do- i composti otticamente attivi ruotano la luce polarizzata sul piano).
@a-cyclohexane-molecule Una versione di quella risposta in termini che un chimico potrebbe capire sarebbe una risposta decente qui (specialmente se puoi evitare qualsiasi meccanica quantistica o matematica).
Una risposta:
Tyberius
2017-10-14 00:28:41 UTC
view on stackexchange narkive permalink

Come accennato nei commenti, la risposta alla tua $ 2 ^ {nd} $ domanda sul perché la rotazione ottica non sia semplicemente annullata nella soluzione a causa di orientamenti casuali è data nella risposta di ManishEarth a Chiralità molecolare e rotazione ottica

Per la tua prima domanda, cercherò di spiegare cosa provoca effettivamente la rotazione quando la luce colpisce una molecola otticamente attiva. Per le molecole chirali, è semplifica il quadro pensare a ciascun enantiomero come una vite di carica negativa che sia destra o sinistra. La luce piana o polarizzata linearmente può sempre essere scomposta in componenti polarizzate circolarmente in senso orario e antiorario. Quando la luce si scontra con la molecola, la componente oraria e quella antioraria interagiscono in modo diverso con il campo elettrico della molecola, portando uno di questi componenti a viaggiare più velocemente dell'altro, il che altera l'angolo del piano di luce polarizzata. Una buona visualizzazione di questo effetto (per un'intera soluzione piuttosto che una singola molecola) può essere trovata su http://cddemo.szialab.org/.

Per inciso a parte, potresti immaginare che, poiché abbiamo una ragionevole comprensione del motivo per cui si verifica la rotazione ottica, dovremmo essere in grado di prevedere, euristicamente o da un calcolo dei primi principi, l'ampiezza e la direzione dell'ottica rotazione per una molecola solo in base alla sua struttura. Finora non è stato così. Non solo ci sono pochissimi modi per indovinare euristicamente la grandezza / direzione, ma i calcoli di meccanica quantistica ad alti livelli di teoria si sono dimostrati incapaci di ottenere anche in modo coerente la direzione corretta della rotazione ottica. Data una particolare molecola chirale, non possiamo ancora prevedere in quale direzione, per non parlare della magnitudine, un particolare enantiomero ruoterà la luce.

Il mio punto principale in questa piccola tangente è che anche qualcosa di apparentemente semplice e in uso così diffuso come la rotazione ottica non è ancora molto ben compreso ed è un'area difficile di ricerca attiva.



Questa domanda e risposta è stata tradotta automaticamente dalla lingua inglese. Il contenuto originale è disponibile su stackexchange, che ringraziamo per la licenza cc by-sa 3.0 con cui è distribuito.
Loading...